01 - Quiz - Making Priority Decisions

Pataasin ang iyong marka sa homework at exams ngayon gamit ang Quizwiz!

A patient with abdominal pain and irregular vaginal bleeding is admitted to the hospital with a suspected ectopic pregnancy. Before actual diagnosis, what is the most appropriate action by the nurse? a. Provide analgesics for pain relief. b. Monitor vital signs, pain, and bleeding frequently. c. Explain the need for frequent blood samples for β-human chorionic gonadotropin monitoring. d. Offer support for the patient's emotional response to the loss of the pregnancy.

b. Ectopic pregnancy is a life-threatening condition. If the fallopian tube ruptures, profuse bleeding can lead to hypovolemic shock. All of the interventions are indicated but the priority is monitoring the vital signs and pain for evidence of bleeding.

During the primary survey the nurse identified asymmetric chest wall movement in the patient. What intervention should the nurse do first? a. Check a central pulse. b. Stabilize the cervical spine. c. Apply direct pressure to the wound. d. Administer bag-mask ventilation with 100% oxygen.

d. Asymmetric chest wall movement may indicate a flail chest, which requires bag-mask ventilation with 100% oxygen and may require intubation. A central pulse is checked and pressure is applied to a wound when there is profuse bleeding. The cervical spine is stabilized if there is any suspicion of a head or neck injury.

An immediate prosthetic fitting during surgery is used for a patient with a traumatic below-theknee amputation. During the immediate postoperative period, what is a priority nursing intervention? a. Monitor the patient's vital signs. b. Assess the incision for hemorrhage. c. Elevate the residual limb on pillows. d. Have the patient flex and extend the knee every hour.

a. Because the device covers the residual limb, the surgical site cannot be directly seen and postoperative hemorrhage is not apparent on dressings, requiring vigilant assessment of vital signs for signs of bleeding. Elevation of the residual limb with an immediate prosthetic fitting is not necessary because the device itself prevents edema formation. Exercises to the leg are not performed in the immediate postoperative period to avoid disruption of ligatures and the suture line.

Following auscultation of the abdomen, what should the nurse's next action be? a. Lightly percuss over all four quadrants b. Have the patient empty his or her bladder c. Inspect perianal and anal areas for color, masses, rashes, and scars d. Perform deep palpation to delineate abdominal organs and masses

a. The abdomen should be assessed in the following sequence: inspection, auscultation, percussion, palpation. The patient should empty his or her bladder before assessment begins.

When obtaining a health history from a patient with cataracts, what is most important for the nurse to ask about the use of? a. Corticosteroids b. Oral hypoglycemic agents c. Antihistamines and decongestants d. β-Adrenergic blocking agents

a. The use of corticosteroids has been associated with the development of cataracts and glaucoma. Use of oral hypoglycemic agents alerts the nurse to the presence of diabetes and risk for diabetic retinopathy. Antihistamine and decongestant drugs may cause eye dryness. β-Adrenergic blocking agents may cause additive effects in patients with glaucoma for whom these medications may be prescribed.

A patient who is postoperative following repair of an AAA has been receiving IV fluids at 125 mL/hr continuously for the last 12 hours. Urine output for the last 4 hours has been 60 mL, 42 mL, 28 mL, and 20 mL, respectively. What is the priority action that the nurse should take? a. Monitor for a couple more hours. b. Contact the physician and report the decrease in urine output. c. Send blood for electrolytes, blood urea nitrogen (BUN), and creatinine. d. Decrease the rate of infusion to prevent blood leakage at the suture line.

b. The decreasing urine output is evidence that either the patient needs volume or there is reduced renal blood flow. The physician will want to be notified as soon as possible of this change in condition and may order laboratory tests. The other options are incorrect.

During care of a patient in myasthenic crisis, maintenance of what is the nurse's first priority for the patient? a. Mobility b. Nutrition c. Respiratory function d. Verbal communication

c. The patient in myasthenic crisis has severe weakness and fatigability of all skeletal muscles, affecting the patient's ability to breathe, swallow, talk, and move. However, the priority of nursing care is monitoring and maintaining adequate ventilation.

On assessment of a central venous access device (CVAD) site, the nurse observes that the transparent dressing is loose along two sides. What should the nurse do immediately? a. Wait and change the dressing when it is due. b. Tape the two loose sides down and document. c. Apply a gauze dressing over the transparent dressing and tape securely. d. Remove the dressing and apply a new transparent dressing using sterile technique.

d. The greatest risk with CVAD is systemic infection. Dressings that are loose should be changed immediately to reduce this risk.

What nursing action is most important for the patient with age-related macular degeneration (AMD)? a. Teach the patient how to use topical eyedrops for treatment of AMD. b. Emphasize the use of vision enhancement techniques to improve what vision is present. c. Encourage the patient to undergo laser treatment to slow the deposit of extracellular debris. d. Explain that nothing can be done to save the patient's vision because there is no treatment for AMD.

. b. The patient with AMD can benefit from low-vision aids despite increasing loss of vision and it is important to promote a positive outlook by not giving patients the impression that "nothing can be done" for them. Laser treatment may help a few patients with choroidal neovascularization and photodynamic therapy is indicated for a small percentage of patients with wet AMD but there is no treatment for the increasing deposit of extracellular debris in the retina.

The nurse finds a patient in bed having a generalized tonic-clonic seizure. During the seizure activity, what actions should the nurse take (select all that apply)? a. Loosen restrictive clothing. b. Turn the patient to the side. c. Protect patient's head from injury. d. Place padded tongue blade between patient's teeth. e. Restrain patient's extremities to prevent soft tissue and bone injury.

a, b, c. The focus is on maintaining a patent airway and preventing patient injury. The nurse should not place objects in the patient's mouth or restrain the patient.

The nurse is teaching a moderately obese woman interventions for the management of obesity. Initially, which strategies will support restricting dietary intake to below energy requirements (select all that apply)? a. Limit alcohol b. Rest when fatigued c. Determine portion sizes d. 1800- to 2200-calorie diet e. Attend Overeaters Anonymous

a, c. To restrict dietary intake so that it is below energy requirements, the moderately obese woman should limit or avoid alcohol intake because it increases caloric intake and has low nutritional value. Portion sizes have increased over the years and are larger than they should be. Teach the patient to determine portion sizes by weight or learn equivalencies such as that a serving of fruit is the size of a baseball. A progressive exercise program will increase energy requirements and a diet with an initial 800- to 1200-calorie limit would decrease calorie intake. Overeaters Anonymous would not restrict dietary intake below energy requirements, although it may offer support for the patient.

Triage the following patient situations that may be present in an emergency department (ED) as 1, 2, 3, 4, or 5 on the Emergency Severity Index. a. A 6-year-old child with a temperature of 103.2°F (39.6°C) b. A 22-year-old woman with asthma in acute respiratory distress c. An infant who has been vomiting for 2 days d. A 50-year-old man with low back pain and spasms e. A 32-year-old woman who is unconscious following an automobile accident f. A 40-year-old woman with rhinitis and a cough g. A 58-year-old man with midsternal chest pain h. A 16-year-old teenager with an angulated forearm following a sports injury

a. 2; b. 1; c. 2; d. 4; e. 1; f. 5; g. 2; h. 3

The nurse notes drainage on the surgical dressing when the patient is transferred from the PACU to the clinical unit. In what order of priority should the nurse do the following actions? Number the options with 1 for the first action and 5 for the last action. a. Reinforce the surgical dressing. b. Change the dressing and assess the wound as ordered. c. Notify the surgeon of excessive drainage type and amount. d. Recall the report from PACU for the number and type of drains in use. e. Note and record the type, amount, and color and odor of the drainage.

a. 2; b. 5; c. 4; d. 1; e. 3. The nurse must be aware of drains, if used, and the type of surgery to help predict the expected drainage. Dressings over surgical sites are initially removed by the surgeon unless otherwise specified and should not be changed, although reinforcing the dressing is appropriate. Some drainage is expected for most surgical wounds and the drainage should be evaluated and recorded to establish a baseline for continuing assessment. The surgeon should be notified of excessive drainage.

A patient with a spinal cord injury suddenly experiences a throbbing headache, flushed skin, and diaphoresis above the level of injury. After checking the patient's vital signs and finding a systolic blood pressure of 210 and a heart rate of 48 bpm, number the following nursing actions in order of priority from highest to lowest (begin with number 1 as first priority). a. Administer ordered prn nifedipine (Procardia). b. Check for bladder distention. c. Document the occurrence, treatment, and response. d. Place call to physician. e. Raise the head of bed (HOB) to 45 degrees or above. f. Loosen tight clothing on the patient.

a. 5; b. 2; c. 6; d. 3; e. 1; f. 4. The patient is experiencing autonomic dysreflexia. The initial response by the nurse should be to elevate the head of bed (HOB) to decrease blood pressure (BP) and then to remove noxious stimulation. Frequently the trigger is bladder distention, which can be dealt with quickly. The physician needs to be notified as soon as possible and, depending on the communication system available to the nurse, he or she should have the call placed. Meanwhile, the nurse should stay with the patient and loosen any restrictive clothing. The physician may order an antihypertensive and documentation should be an accurate and thorough description of the entire episode.

A patient hospitalized for evaluation of unstable angina experiences severe chest pain and calls the nurse. Prioritize the interventions below from 1 (highest priority) to 6 (lowest priority). The appropriate medical orders and protocols are available to the nurse. a. Notify the physician b. Obtain a 12-lead ECG c. Check the patient's vital signs d. Administer oxygen per nasal cannula e. Perform a focused assessment of the chest f. Assess pain (PQRST) and medicate as ordered

a. 6; b. 4; c. 3; d. 1; e. 5; f. 2. A patient having chest pain needs to have the pain assessed and relieved as quickly as possible. The administration of oxygen may help to relieve the pain. Following an assessment of the pain, medication may be administered. It is important to know if the pain is accompanied by a change in vital signs and if any other manifestations or ECG changes exist before the report is given to the physician.

A patient is undergoing a contrast computed tomography (CT) of the spleen. What is most important for the nurse to ask the patient about before the test? a. Iodine sensitivity b. Prior blood transfusions c. Phobia of confined spaces d. Internal metal implants or appliances

a. A contrast CT scan involves the use of an iodine-based dye that could cause a reaction if the patient is sensitive to iodine. Metal implants or internal appliances and claustrophobia should be determined before magnetic resonance imaging (MRI). Prior blood transfusions are not a factor in this diagnostic test.

A patient's rhythm strip indicates a normal HR and rhythm with normal P waves and QRS complexes, but the PR interval is 0.26 second. What is the most appropriate action by the nurse? a. Continue to assess the patient. b. Administer atropine per protocol. c. Prepare the patient for synchronized cardioversion. d. Prepare the patient for placement of a temporary pacemaker.

a. A rhythm pattern that is normal except for a prolonged P-R interval is characteristic of a firstdegree heart block. First-degree heart blocks are not treated but are observed for progression to higher degrees of heart block. Atropine is administered for bradycardia. Synchronized cardioversion is used for atrial fibrillation with a rapid ventricular response or supraventricular tachycardia (SVT). Pacemakers are used for higher-degree heart blocks.

The nurse is caring for a patient receiving 1000 mL of parenteral nutrition solution over 24 hours. When it is time to change the solution, 150 mL remain in the bottle. What is the most appropriate action by the nurse? a. Hang the new solution and discard the unused solution. b. Open the IV line and rapidly infuse the remaining solution. c. Notify the health care provider for instructions regarding the infusion rate. d. Wait to change the solution until the remaining solution infuses at the prescribed rate.

a. Bacterial growth occurs at room temperature in nutritional solutions. Therefore solutions must not be infused for longer than 24 hours. Remaining solution should be discarded. Speeding up the solution may cause hyperglycemia and should not be done. The health care provider does not need to be notified.

During admission to the emergency department, a patient with chronic alcoholism is intoxicated and very disoriented and confused. Which drug will the nurse administer first? a. IV thiamine b. IV benzodiazepines c. IV haloperidol (Haldol) d. IV naloxone (Narcan) in normal saline

a. Because Wernicke's encephalopathy resulting from a thiamine deficiency is a possibility with chronic alcoholism, IV thiamine is often administered to intoxicated patients to prevent the development of Korsakoff's psychosis. Thiamine should be given before any glucose solutions are administered because glucose can precipitate Wernicke's Encephalopathy. Benzodiazepines may be used for sedation and to minimize withdrawal symptoms but would not be given before thiamine, and haloperidol could be used if hallucinations occur.

Two days following a self-managed hypoglycemic episode at home, the patient tells the nurse that his blood glucose levels since the episode have been between 80 and 90 mg/dL. Which is the best response by the nurse? a. "That is a good range for your glucose levels." b. "You should call your health care provider because you need to have your insulin increased." c. "That level is too low in view of your recent hypoglycemia and you should increase your food intake." d. "You should take only half your insulin dosage for the next few days to get your glucose level back to normal."

a. Blood glucose levels of 80 to 90 mg/dL (4.4 to 5 mmol/L) are within the normal range and are desired in the patient with diabetes, even following a recent hypoglycemic episode. Hypoglycemia is often caused by a single event, such as skipping a meal, taking too much insulin, or vigorous exercise. Once corrected, normal glucose control should be maintained.

A patient with a gastric outlet obstruction has been treated with NG decompression. After the first 24 hours, the patient develops nausea and increased upper abdominal bowel sounds. What is the best action by the nurse? a. Check the patency of the NG tube. b. Place the patient in a recumbent position. c. Assess the patient's vital signs and circulatory status. d. Encourage the patient to deep breathe and consciously relax.

a. If symptoms of gastric outlet obstruction, such as nausea, vomiting, and stomach distention, occur while the patient is on NPO status or has an NG tube, the patency of the NG tube should be assessed. A recumbent position should not be used in a patient with a gastric outlet obstruction because it increases abdominal pressure on the stomach and vital signs and circulatory status assessment are important if hemorrhage or perforation is suspected. Deep breathing and relaxation may help some patients with nausea but not when stomach contents are obstructed from flowing into the small intestine.

When a patient is admitted to the ED following a head injury, what should be the nurse's first priority in management of the patient once a patent airway is confirmed? a. Maintain cervical spine precautions. b. Monitor for changes in neurologic status. c. Determine the presence of increased ICP. d. Establish IV access with a large-bore catheter.

a. In addition to monitoring for a patent airway during emergency care of the patient with a head injury, the nurse must always assume that a patient with a head injury may have a cervical spine injury. Maintaining cervical spine precautions in all assessment and treatment activities with the patient is essential to prevent additional neurologic damage.

A patient with a gunshot wound to the abdomen complains of increasing abdominal pain several hours after surgery to repair the bowel. What action should you take first? a. Take the patient's vital signs. b. Notify the health care provider. c. Position the patient with the knees flexed. d. Determine the patient's IV intake since the end of surgery.

a. It is likely that the patient could be developing a peritonitis, which could be life-threatening, and assessment of vital signs for hypovolemic shock should be done to report to the health care provider. If an IV line is not in place, it should be inserted and pain may be eased by flexing the knees.

After endotracheal intubation and mechanical ventilation have been started, a patient in respiratory failure becomes very agitated and is breathing asynchronously with the ventilator. What is it most important for the nurse to do first? a. Evaluate the patient's pain level, ABGs, and electrolyte values b. Sedate the patient to unconsciousness to eliminate patient awareness c. Administer the PRN vecuronium (Norcuron) to promote synchronous ventilations d. Slow the rate of ventilations provided by the ventilator to allow for spontaneous breathing by the patient

a. It is most important to assess the patient for the cause of the restlessness and agitation (e.g., pain, hypoxemia, electrolyte imbalances) and treat the underlying cause before sedating the patient. Although sedation, analgesia, and neuromuscular blockade are often used to control agitation and pain, these treatments may contribute to prolonged ventilator support and hospital days.

A 75-year-old patient who is breathing room air has the following arterial blood gas (ABG) results: pH 7.40, PaO2 74 mm Hg, SaO2 92%, PaCO2 40 mm Hg. What is the most appropriate action by the nurse? a. Document the results in the patient's record. b. Repeat the ABGs within an hour to validate the findings. c. Encourage deep breathing and coughing to open the alveoli. d. Initiate pulse oximetry for continuous monitoring of the patient's oxygen status.

a. Normal findings in arterial blood gases (ABGs) in the older adult include a small decrease in PaO2 and SaO2 but normal pH and PaCO2. No interventions are necessary for these findings. Usual PaO2 levels are expected in patients 60 years of age or younger.

A patient has atrial fibrillation and develops an acute arterial occlusion at the iliac artery bifurcation. What are the six Ps of acute arterial occlusion the nurse may assess in this patient that require immediate notification of the physician?

a. Pain; b. pallor; c. pulselessness; d. paresthesia; e. paralysis; f. poikilothermia. The physician requires immediate notification to begin immediate intervention to prevent tissue necrosis and gangrene.

After receiving the assigned patients for the day, the nurse determines that stress-relieving interventions are a priority for which patient? a. The man with peptic ulcer disease b. The newly admitted woman with cholecystitis c. The man with a bacterial exacerbation of chronic bronchitis d. The woman who is 1 day postoperative for knee replacement

a. Peptic ulcer disease is one of several disorders with a known stress component. Although many patients have stress related to a health problem, stress-relieving interventions are always indicated for patients with diseases in which stress contributes to the problem.

The nurse reviews the following vital signs recorded by an unlicensed assistive personnel (UAP) on a patient with acute decompensated heart failure: BP 98/60, HR 102 bpm, RR 24, Temp 98.2°F (36.7° C), SpO2 84% on 2 L/min via nasal cannula. a. Which of these findings is of highest priority? b. What should the nurse do next?

a. SpO2 of 84% on 2 L/min via nasal cannula indicates impaired oxygen saturation. The patient is having trouble with gas exchange. Airway and breathing are the priority (follow ABCs). b. The nurse should place the patient in high Fowler's position, assess the patient immediately, recheck SpO2, auscultate breath sounds, assess level of consciousness (LOC), check the oxygen connection and rate setting (2 L/min), and talk with the patient about her or his breathing.

A postoperative patient has a nursing diagnosis of pain related to effects of medication and decreased GI motility as evidenced by abdominal pain and distention and inability to pass flatus. Which nursing intervention is most appropriate for this patient? a. Ambulate the patient more frequently. b. Assess the abdomen for bowel sounds. c. Place the patient in high Fowler's position. d. Withhold opioids because they decrease bowel motility.

a. The abdominal pain and distention that occur from the decreased motility of the bowel should be treated with increased ambulation and frequent position changes to increase peristalsis. If the pain is severe, cholinergic drugs, rectal tubes, or application of heat to the abdomen may be prescribed. Assessment of bowel sounds is not an intervention to relieve the pain and a high Fowler's position is not indicated. Opioids may still be necessary for pain control and motility can be increased by other means.

A patient in the PACU has emergence delirium manifested by agitation and thrashing. What should the nurse assess for first in the patient? a. Hypoxemia b. Neurologic injury c. Distended bladder d. Cardiac dysrhythmias

a. The most common cause of emergence delirium is hypoxemia and initial assessment should evaluate respiratory function. When hypoxemia is ruled out, other causes, such as a distended bladder, pain, and fluid and electrolyte disturbances, should be considered. Delayed awakening may result from neurologic injury and cardiac dysrhythmias most often result from specific respiratory, electrolyte, or cardiac problems

A patient is admitted to the emergency department with a possible cervical spinal cord injury following an automobile crash. During admission of the patient, what is the highest priority for the nurse? a. Maintaining a patent airway b. Maintaining immobilization of the cervical spine c. Assessing the patient for head and other injuries d. Assessing the patient's motor and sensory function

a. The need for a patent airway is the first priority for any injured patient and a high cervical injury may decrease the gag reflex and the ability to maintain an airway as well as the ability to breathe. Maintaining cervical stability is then a consideration, along with assessing for other injuries and the patient's neurologic status.

When planning care for the patient with trigeminal neuralgia, which patient outcome should the nurse set as the highest priority? a. Relief of pain b. Protection of the cornea c. Maintenance of nutrition d. Maintenance of positive body image

a. The pain of trigeminal neuralgia is excruciating and it may occur in clusters that continue for hours. The condition is considered benign with no major effects except the pain. Corneal exposure is a problem in Bell's palsy or it may occur following surgery for the treatment of trigeminal neuralgia. Maintenance of nutrition is important but not urgent because chewing may trigger trigeminal neuralgia and patients then avoid eating. Except during an attack, there is no change in facial appearance in a patient with trigeminal neuralgia and body image is more disturbed in response to the paralysis typical of Bell's palsy.

Following electrohydraulic lithotripsy for treatment of renal calculi, the patient has a nursing diagnosis of risk for infection related to the introduction of bacteria following manipulation of the urinary tract. What is the most appropriate nursing intervention for this patient? a. Monitor for hematuria. b. Encourage fluid intake of 3 L/day. c. Apply moist heat to the flank area. d. Strain all urine through gauze or a special strainer.

b. A high fluid intake maintains dilute urine, which decreases bacterial concentration in addition to washing stone fragments and expected blood through the urinary system following lithotripsy. High urine output also prevents supersaturation of minerals. Moist heat to the flank may be helpful to relieve muscle spasms during renal colic and all urine should be strained in patients with renal stones to collect and identify stone composition but these are not related to infection

A patient is admitted to the emergency department with an acute asthma attack. Which patient assessment is of greatest concern to the nurse? a. The presence of a pulsus paradoxus b. Markedly diminished breath sounds with no wheezing c. Use of accessory muscles of respiration and a feeling of suffocation d. A respiratory rate of 34 and increased pulse and blood pressure

b. Diminished or absent breath sounds may indicate a significant decrease in air movement resulting from exhaustion and an inability to generate enough muscle force to ventilate and is an ominous sign. The other symptoms are expected in an asthma attack.

A patient with symptomatic mitral valve prolapse has atrial and ventricular dysrhythmias. In addition to monitoring for decreased cardiac output related to the dysrhythmias, what is an important nursing intervention related to the dysrhythmias identified by the nurse? a. Monitor breathing pattern related to hypervolemia. b. Encourage calling for assistance when getting out of bed. c. Give sleeping pills to decrease paroxysmal nocturnal dyspnea. d. Teach the patient exercises to prevent recurrence of dysrhythmias.

b. Dysrhythmias frequently cause palpitations, lightheadedness, and dizziness and the patient should be carefully attended to to prevent falls. Hypervolemia and paroxysmal nocturnal dyspnea (PND) would be apparent in the patient with heart failure. Exercises will not prevent dysrhythmias.

Results of an audiometry indicate that a patient has a 10-dB hearing loss at 8000 Hz. What is the most appropriate action by the nurse? a. Encourage the patient to start learning to lip-read b. Speak at a normal speed and volume with the patient c. Avoid words in conversation that have many high-pitched consonants d. Discuss the advantages and disadvantages of various hearing aids with the patient

b. Hearing is most sensitive between 500 and 4000 Hz and a 10-dB loss is not significant at 8000 Hz. A 40- to 45-dB loss in the frequency between 4000 and 8000 Hz will cause difficulty in distinguishing highpitched consonants. A hearing aid is rarely recommended for a hearing loss of less than 26 dB, and problems in everyday communication situations occur only when the thresholds are 25 dB or higher.

A terminally ill patient is unresponsive and has cold, clammy skin with mottling on the extremities. The patient's husband and two grown children are arguing at the bedside about where the patient's funeral should be held. What should the nurse do first? a. Ask the family members to leave the room if they are going to argue. b. Take the family members aside and explain that the patient may be able to hear them. c. Tell the family members that this decision is premature because the patient has not yet died. d. Remind the family that this should be the patient's decision and to ask her if she regains consciousness.

b. Hearing is often the last sense to disappear with declining consciousness and conversations can distress patients even when they appear unresponsive. Conversation around unresponsive patients should never be other than that which one would maintain if the patients were alert.

When medications are used in the treatment of obesity, what is most important for the nurse to teach the patient? a. Over-the-counter (OTC) diet aids are safer than other agents and can be useful in controlling appetite. b. Drugs should be used only as adjuncts to a diet and exercise program as treatment for a chronic condition. c. All drugs used for weight control are capable of altering central nervous system (CNS) function and should be used with caution. d. The primary effect of the medications is psychologic, controlling the urge to eat in response to stress or feelings of rejection.

b. Medications are used only as adjuncts to diet and exercise programs in the treatment of obesity. Drugs do not cure obesity; without changes in food intake and physical activity, weight gain will occur when the medications are discontinued. The medications used work in a variety of ways to control appetite but over-the-counter (OTC) drugs are probably the least effective and most abused of these drugs.

What is the priority nursing intervention during the management of the patient with pheochromocytoma? a. Administering IV fluids b. Monitoring blood pressure c. Administering β-adrenergic blockers d. Monitoring intake and output and daily weights

b. Pheochromocytoma is a catecholamine-producing tumor of the adrenal medulla, which may cause severe, episodic hypertension; severe, pounding headache; and profuse sweating. Monitoring for a dangerously high BP before surgery is critical, as is monitoring for BP fluctuations during medical and surgical treatment.

A patient with stage 2 hypertension who is taking hydrochlorothiazide (HydroDiuril) and lisinopril (Prinivil) has prazosin (Minipress) added to the medication regimen. What is most important for the nurse to teach the patient to do? a. Weigh every morning to monitor for fluid retention b. Change position slowly and avoid prolonged standing c. Use sugarless gum or candy to help relieve dry mouth d. Take the pulse daily to note any slowing of the heart rate

b. Prazosin is an α-adrenergic blocker that causes dilation of arterioles and veins and causes orthostatic hypotension. The patient may feel dizzy, weak, and faint when assuming an upright position after sitting or lying down and should be taught to change positions slowly, avoid standing for long periods, do leg exercises to increase venous return, and lie or sit down when dizziness occurs. Direct-acting vasodilators often cause fluid retention; dry mouth occurs with diuretic use, although orthostatic hypotension may occur with hydrochlorothiazide as well; and centrally acting α- and βblockers may cause bradycardia.

Following a dilation and curettage (D&C), what complication is most important for the nurse to assess the patient for? a. Infection b. Hemorrhage c. Urinary retention d. Perforation of the bladder

b. The risk for bleeding is increased following a dilation and curettage (D&C) because the endometrial lining is scraped and injury to the uterus can occur. The nurse should closely assess the amount of bleeding with frequent pad checks for the first 24 hours. Infection following D&C is uncommon and the urinary system is not affected.

Upon admission of a patient to the PACU, the nurse's priority assessment is a. vital signs. b. surgical site. c. respiratory adequacy. d. level of consciousness.

c. Physiologic status of the patient is always prioritized with regard to airway, breathing, and circulation, and respiratory adequacy is the first assessment priority of the patient on admission to the PACU from the operating room. Following assessment of respiratory function, cardiovascular, neurologic, and renal function should be assessed as well as the surgical site.

A patient is brought to the ED following a skiing accident after which he was not found for several hours. He is rigid and has slowed respiratory and heart rates. What should the nurse do during the primary assessment of the patient? a. Initiate active core rewarming interventions. b. Monitor the core temperature via the axillary route. c. Manage and maintain ABCs (airway, breathing, circulation). d. Expose the patient to check for areas of frostbite and other injuries.

c. Rigidness, bradycardia, and slowed respiratory rate are signs of moderate hypothermia. The ABCs are the initial priority. Active core rewarming is indicated for moderate to severe hypothermia. Axillary temperatures are inadequate to monitor core temperature, so esophageal, rectal, or indwelling urinary catheter thermometers are used. The patient should be assessed for other injuries but should not be exposed, to prevent further loss of heat.

Although his oxygen saturation is above 92%, an orally intubated, mechanically ventilated patient is restless and very anxious. What intervention should be used first to decrease the risk of accidental extubation? a. Obtain an order and apply soft wrist restraints. b. Remind the patient that he needs the tube inserted to breathe. c. Administer sedatives and have a caregiver stay with the patient. d. Move the patient to an area close to the nurses' station for closer observation.

c. Sedation may be appropriate. As well, having someone the patient knows at the bedside talking to him and reassuring him may decrease his anxiety and calm him. Restraints have not been shown to be an absolute deterrent to self-extubation and the patient will need ongoing and frequent assessment of need. Reminding the patient of the need for the tube may help but it may not be enough to prevent him from pulling out the tube if he becomes extremely anxious. Moving the patient near the nurses' station will not be enough to prevent self-extubation since it can be done quickly.

Twenty-four hours after a below-the-knee amputation, a patient uses the call system to tell the nurse that his dressing (a compression bandage) has fallen off. What is the first action that the nurse should take? a. Apply ice to the site. b. Cover the incision with dry gauze. c. Reapply the compression dressing. d. Elevate the extremity on a couple of pillows.

c. The compression dressing or bandage supports the soft tissues, reduces edema, hastens healing, minimizes pain, and promotes residual limb shrinkage. If the dressing is left off, edema will form quickly and may delay rehabilitation. Elevation and ice will not be as effective at preventing the edema that will form. Dressing the incision with dry gauze will not provide the benefits of a compression dressing.

While caring for a patient who is at the nadir of chemotherapy, the nurse establishes the highest priority for nursing actions related to a. diarrhea. b. grieving. c. risk for infection. d. inadequate nutritional intake.

c. The nadir is the point of the lowest blood counts after chemotherapy is started and it is the time when the patient is most at risk for infection. Because infection is the most common cause of morbidity and death in cancer patients, identification of risk and interventions to protect the patient are of the highest priority. The other problems will be treated but they are not the priority.

The patient has hepatic encephalopathy. What is a priority nursing intervention to keep the patient safe? a. Turn the patient every 3 hours. b. Encourage increasing ambulation. c. Assist the patient to the bathroom. d. Prevent constipation to reduce ammonia production.

c. The patient may not be oriented or able to walk to the bathroom alone because of hyperreflexia, asterixis, or decreased motor coordination. Turning should be done every 2 hours to prevent skin breakdown. Activity is limited to decrease ammonia as a by-product of protein metabolism. Although constipation will be prevented, it will not keep the patient safe.

During care of patients, what is the most important precaution for preventing transmission of infections? a. Wearing face and eye protection during routine daily care of the patient b. Wearing nonsterile gloves when in contact with body fluids, excretions, and contaminated items c. Wearing a gown to protect the skin and clothing during patient care activities likely to soil clothing d. Hand washing after touching fluids and secretions and removing gloves, as well as between patient contacts

d. Hand washing is the most important factor in preventing infection transmission and is recommended before and after the use of gloves by the Centers for Disease Control and Prevention for all types of isolation precautions in health care facilities.

A patient with acute pancreatitis is experiencing hypovolemic shock. Which initial orders for the patient will the nurse implement first? a. Start 1000 mL of normal saline at 500 mL/hr. b. Obtain blood cultures before starting IV antibiotics. c. Draw blood for hematology and coagulation factors. d. Administer high-flow oxygen (100%) with a non-rebreather mask.

d. In every type of shock there is a deficiency of oxygen to the cells and high-flow oxygen therapy is indicated. Fluids would be started next, blood cultures would be done before any antibiotic therapy, and laboratory specimens then could be drawn.

A patient has fallen in the bathroom of the hospital room and reports pain in the upper right arm and elbow. Before splinting the injury, the nurse knows that the priority management of a possible fracture should include which action? a. Elevation of the arm b. Application of ice to the site c. Notification of the health care provider d. Neurovascular checks below the site of the injury

d. Pulses, sensation, and motor function distal to the injury should be checked before and after splinting to assess for nerve or vascular damage and documented to avoid doubts about whether a problem discovered later was missed during the original examination or was caused by the treatment. Elevation of the limb and application of ice should be instituted after the extremity is splinted.

A patient with acute pericarditis has a nursing diagnosis of pain related to pericardial inflammation. What is the best nursing intervention for the patient? a. Administer opioids as prescribed on an around-the-clock schedule. b. Promote progressive relaxation exercises with the use of deep, slow breathing. c. Position the patient on the right side with the head of the bed elevated 15 degrees. d. Position patient in Fowler's position with a padded over-the-bed table for the patient to lean on.

d. Relief from pericardial pain is often obtained by sitting up and leaning forward. Pain is increased by lying flat. The pain has a sharp, pleuritic quality that changes with respiration and patients take shallow breaths. Antiinflammatory medications may also be used to help control pain but opioids are not usually indicated.

Which medication should the nurse anticipate being used first in the emergency department for relief of severe respiratory distress related to asthma? a. Prednisone orally b. Ipratopium inhaler c. Fluticasone inhaler d. Albuterol nebulizer

d. The albuterol nebulizer will rapidly cause bronchodilation and be easier to use in an emergency situation than an inhaler. It will be used every 20 minutes to 4 hours as needed. The ipratropium inhaler could be used if the patient does not tolerate the short-acting β2-adrenergic agonists (SABA) but its onset is slower than albuterol. Inhaled or oral corticosteroids will be used to decrease the inflammation and provide better symptom control after the emergency situation is over.

An 18-year-old female patient with anorexia nervosa is admitted to the hospital for treatment. On admission she weighs 82 lb (37 kg) and is 5 ft, 3 in (134.6 cm). Her laboratory test results include the following: K+ 2.8 mEq/L (2.8 mmol/L), Hgb 8.9 g/dL (89 g/L), and BUN 64 mg/dL (22.8 mmol/L). In planning care for the patient, the nurse gives the highest priority to which of the following nursing diagnoses? a. Risk for injury related to dizziness and weakness resulting from anemia b. Imbalanced nutrition: less than body requirements related to inadequate food intake c. Risk for impaired urinary elimination related to elevated BUN resulting from renal failure d. Risk for decreased cardiac output (CO) related to dysrhythmias resulting from hypokalemia

d. The potential life-threatening cardiac complications related to the hypokalemia are the most important immediate considerations in the patient's care. The other nursing diagnoses are important for the patient's care but do not pose the immediate risk that the hypokalemia does.

During an acute exacerbation of mild COPD, the patient is severely short of breath and the nurse identifies a nursing diagnosis of ineffective breathing pattern related to obstruction of airflow and anxiety. What is the best action by the nurse? a. Prepare and administer routine bronchodilator medications. b. Perform chest physiotherapy to promote removal of secretions. c. Administer oxygen at 5 L/min until the shortness of breath is relieved. d. Position the patient upright with the elbows resting on the over-the-bed table.

d. The tripod position with an elevated backrest and supported upper extremities to fix the shoulder girdle maximizes respiratory excursion and an effective breathing pattern. Staying with the patient and encouraging pursed lip breathing also helps. Bronchodilators may help but can also increase nervousness and anxiety; rescue inhalers would be used before routine bronchodilators. Postural drainage is not tolerated by a patient in acute respiratory distress and oxygen is titrated to an effective rate based on ABGs because of the possibility of carbon dioxide narcosis.

A patient in the last stages of life is experiencing shortness of breath and air hunger. Based on practice guidelines, what is the most appropriate action by the nurse? a. Administer oxygen. b. Administer bronchodilators. c. Administer antianxiety agents. d. Use any methods that make the patient more comfortable.

d. There currently are no clinical practice guidelines to relieve the shortness of breath and air hunger that often occur at the end of life. The principle of beneficence would encourage any of the options to be tried, based on knowing that whatever gives the patient the most relief should be used.

During evaluation and treatment of gonorrhea in a young man at the health clinic, what is most important for the nurse to question the patient about? a. A prior history of STIs b. When the symptoms began c. The date of his last sexual activity d. The names of his recent sexual partners

d. All sexual contacts of patients with gonorrhea must be notified, evaluated, and treated for STIs. The other information may be helpful in diagnosis and treatment but the nurse must try to identify the patient's sexual partners.

The nurse is preparing to administer a blood transfusion. Number the actions in order of priority (1 is first priority action; 10 is last priority action). a. Verify the order for the transfusion. b. Ensure that the patient has a patent 18-gauge IV. c. Prime the transfusion tubing and filter with normal saline. d. Verify that the physician has discussed risks, benefits, and alternatives with the patient. e. Obtain the blood product from the blood bank. f. Ask another licensed person (nurse or MD) to assist in verifying the product identification and the patient identification. g. Document outcomes in the patient record. Document vital signs, names of personnel, and starting and ending times. h. Adjust the infusion rate and continue to monitor the patient every 30 minutes for up to an hour after the product is infused. i. Infuse the first 50 mL over 15 minutes, staying with the patient. j. Obtain the patient's vital signs before starting the transfusion.

a. 1; b. 3; c. 4; d. 2; e. 5; f. 6; g. 10; h. 9; i. 8; j. 7.

The patient is admitted from a nearby park with an apparent anaphylactic reaction to a bee sting. He is experiencing dyspnea and hypotension with swelling at the site. Number the following actions in the order of priority that the nurse should implement for this patient. a. Provide oxygen b. Remove the stinger c. Ensure a patent airway d. Prepare to administer epinephrine e. Start IV for fluid and medication access f. Position patient flat with the legs elevated g. Anticipate intubation with severe respiratory distress h. Have diphenhydramine (Benadryl) and nebulized albuterol available

a. 2; b. 4; c. 1; d. 6; e. 5; f. 3; g. 7; h, 8. Airway is always first and oxygen will help with the dyspnea. Knowing that the patient has hypotension leads to placing the patient recumbent with elevated legs. Knowing that the patient has a bee sting, the stinger will be removed next. Then an IV is started. Anticipation of intubation with severe respiratory distress is also needed after the initial interventions.Having diphenhydramine and nebulized albuterol as well as methylprenisolone IV available is important, as they may be needed.

Number the following actions in the order they should be done in the emergency management of a burn of any type, beginning with number 1 for the first action. a. Establish and maintain an airway. b. Assess for other associated injuries. c. Establish an IV line with a large-gauge needle. d. Remove the patient from the burn source and stop the burning process.

a. 2; b. 4; c. 3; d. 1. The first intervention in emergency management of a burn injury is to remove the burn source and stop the burning process. Airway maintenance would be second, then establishing IV access, followed by assessing for other injuries.

The nurse has received the change-of-shift report on his group of patients. Indicate the priority order in which the nurse should see these patients. a. A 40-year-old female who is returning from the postanesthesia care unit (PACU) following surgical debridement of her back and legs b. A 76-year-old male with partial-thickness burns of his arms and abdomen who is complaining of severe pain c. A 62-year-old female who was just admitted following partial-thickness burns to her anterior chest, face, and neck d. An 18-year-old male with full-thickness burns of his lower extremities who is refusing to go for his scheduled dressing change

a. 3; b. 2; c. 1; d. 4. Face and neck burns are frequently associated with airway inhalation. Therefore this patient requires airway assessment (priority = ABCs). Severe pain would be the next priority (high physiologic need). The patient returning from the PACU will need to be seen soon to assess vital signs, level of consciousness, IV fluids, and wounds. However, at the current time the transport personnel should be with her. The 18-year-old is not at risk related to ABCs and it will probably take a few minutes to talk with him about why he doesn't want to go for the dressing change.

A patient's tracheostomy tube becomes dislodged with vigorous coughing. What should be the nurse's first action? a. Attempt to replace the tube. b. Notify the health care provider. c. Place the patient in high Fowler position. d. Ventilate the patient with a manual resuscitation bag until the health care provider arrives.

a. If a tracheostomy tube is dislodged, the nurse should immediately attempt to replace the tube by grasping the retention sutures (if available) and spreading the opening. The obturator is inserted in the replacement tube, watersoluble lubricant is applied to the tip, and the tube is inserted in the stoma at a 45-degree angle to the neck. The obturator is immediately removed to provide an airway. If the tube cannot be reinserted, the health care provider should be notified and the patient should be assessed for the level of respiratory distress, positioned in semi-Fowler position, and ventilated with a manual resuscitation bag (MRB) only if necessary until assistance arrives.

Number the following nursing actions related to care of the patient's central venous access device (CVAD) in the correct order to complete these actions. Number 1 is the first action and number 8 is the last action. a. Perform hand hygiene b. Flush each line with 10 mL of normal saline c. Use strict sterile technique to change the dressing d. Clamp unused lines after flushing if not using positive pressure valve caps e. Assess the CVAD insertion site for redness, edema, warmth, drainage, and pain f. Use friction to cleanse the CVAD insertion site with chlorhexidine-based preparation g. Turn patient's head to the side away from the CVAD insertion site when changing the caps h. Obtain chest x-ray results to verify placement of catheter in distal end of superior vena cava

a. 3; b. 7; c. 4; d. 8; e. 2; f. 5; g. 6; h. 1. The first nursing action to be completed after a CVAD is inserted and before it is used is to ensure proper placement with a chest x-ray. Assessments, flushing, dressing changes, and cap changes are completed according to facility policies but hand hygiene must be completed before manipulating the CVAD to prevent infection. Strict sterile technique is used with dressing and cap changes as well as having the patient turn his or her face away from the insertion site to avoid contamination.

What is the priority of management of the near-drowning patient? a. Correction of hypoxia b. Correction of acidosis c. Maintenance of fluid balance d. Prevention of cerebral edema

a. Airway and oxygenation are the first priorities. A lifethreatening consequence of neardrowning of any type is hypoxia from fluid-filled and poorly ventilated alveoli. Correction of metabolic acidosis occurs with effective ventilation and oxygenation. Lactated Ringer's solution or normal saline solution is started to manage fluid balance and mannitol or furosemide may be used to treat free water and cerebral edema.

Two days after undergoing pelvic surgery, a patient develops marked dyspnea and anxiety. What is the first action that the nurse should take? a. Raise the head of the bed. b. Notify the health care provider. c. Take the patient's pulse and blood pressure. d. Determine the patient's SpO2 with an oximeter.

a. All of the activities are correct but the first thing to do is to raise the head of the bed to promote respiration in the patient who is dyspneic. The health care provider would not be called until the nurse has assessment data relating to vital signs, pulse oximetry, and any other patient complaints.

A patient has ICP monitoring with an intraventricular catheter. What is a priority nursing intervention for the patient? a. Aseptic technique to prevent infection b. Constant monitoring of ICP waveforms c. Removal of CSF to maintain normal ICP d. Sampling CSF to determine abnormalities

a. An intraventricular catheter is a fluid-coupled system that can provide direct access for microorganisms to enter the ventricles of the brain and aseptic technique is a very high nursing priority to decrease the risk for infection. Constant monitoring of ICP waveforms is not usually necessary and removal of CSF for sampling or to maintain normal ICP is done only when specifically ordered.

While receiving a unit of packed RBCs, the patient develops chills and a temperature of 102.2°F (39°C). What is the priority action for the nurse to take? a. Stop the transfusion and instill normal saline. b. Notify the health care provider and the blood bank. c. Add a leukocyte reduction filter to the blood administration set. d. Recognize this as a mild allergic transfusion reaction and slow the transfusion.

a. Chills and fever are symptoms of an acute hemolytic or febrile transfusion reaction and if these develop, the nurse should stop the transfusion, infuse saline through the IV line, notify the health care provider and blood bank immediately, recheck the ID tags, and monitor vital signs and urine output. The addition of a leukocyte reduction filter may prevent a febrile reaction but is not helpful once the reaction has occurred. Mild and transient allergic reactions indicated by itching and hives might permit restarting the transfusion after treatment with antihistamines.

A patient with diabetes calls the clinic because she is experiencing nausea and flu-like symptoms. Which advice from the nurse will be the best for this patient? a. Administer the usual insulin dosage. b. Hold fluid intake until the nausea subsides. c. Come to the clinic immediately for evaluation and treatment. d. Monitor the blood glucose every 1 to 2 hours and call if it rises over 150 mg/dL (8.3 mmol/L).

a. During minor illnesses, the patient with diabetes should continue drug therapy and food intake. Insulin is important because counter regulatory hormones may increase blood glucose during the stress of illness. Food or a carbohydrate liquid substitution is important because during illness the body requires extra energy to deal with the stress of the illness. Blood glucose monitoring should be done every 4 hours and the health care provider should be notified if the level is greater than 240 mg/dL (13.9 mmol/L) or if fever, ketonuria, or nausea and vomiting occur.

A patient on a medical unit has a potassium level of 6.8 mEq/L. What is the priority action that the nurse should take? a. Place the patient on a cardiac monitor. b. Check the patient's blood pressure (BP). c. Instruct the patient to avoid high-potassium foods. d. Call the lab and request a redraw of the lab to verify results.

a. Dysrhythmias may occur with an elevated potassium level and are potentially lethal. Monitor the rhythm while contacting the physician or calling the rapid response team. Vital signs should be checked. Depending on the patient's history and cause of increased potassium, instruct the patient about dietary sources of potassium; however, this would not help at this point. The nurse may want to recheck the value but until then the heart rhythm needs to be monitored.

During the immediate postoperative care of a recipient of a kidney transplant, what should the nurse expect to do? a. Regulate fluid intake hourly based on urine output. b. Monitor urine-tinged drainage on abdominal dressing. c. Medicate the patient frequently for incisional flank pain. d. Remove the urinary catheter to evaluate the ureteral implant.

a. Fluid and electrolyte balance is critical in the transplant recipient patient, especially because diuresis often begins soon after surgery. Fluid replacement is adjusted hourly based on kidney function and urine output. Urine-tinged drainage on the abdominal dressing may indicate leakage from the ureter implanted into the bladder and the health care provider should be notified. The donor patient may have a flank or laparoscopic incision(s) where the kidney was removed. The recipient has an abdominal incision where the kidney was placed in the iliac fossa. The urinary catheter is usually used for 2 to 3 days to monitor urine output and kidney function.

A patient with ulcerative colitis has a total proctocolectomy with formation of a terminal ileum stoma. What is the most important nursing intervention for this patient postoperatively? a. Measure the ileostomy output to determine the status of the patient's fluid balance. b. Change the ileostomy appliance every 3 to 4 hours to prevent leakage of drainage onto the skin. c. Emphasize that the ostomy is temporary and the ileum will be reconnected when the large bowel heals. d. Teach the patient about the high-fiber, low-carbohydrate diet required to maintain normal ileostomy drainage.

a. Initial output from a newly formed ileostomy may be as high as 1500 to 2000 mL daily and intake and output must be accurately monitored for fluid and electrolyte imbalance. Ileostomy bags may need to be emptied every 3 to 4 hours but the appliance should not be changed for several days unless there is leakage onto the skin. A terminal ileum stoma is permanent and the entire colon has been removed. A return to a normal, presurgical diet is the goal for the patient with an ileostomy, with restrictions based only on the patient's individual tolerances.

When caring for a patient with metabolic syndrome, what should the nurse give the highest priority to teaching the patient about? a. Achieving a normal weight b. Performing daily aerobic exercise c. Eliminating red meat from the diet d. Monitoring the blood glucose periodically

a. Metabolic syndrome is a cluster of abnormalities that include elevated glucose levels, abdominal obesity, elevated blood pressure, high levels of triglycerides, and low levels of highdensity lipoproteins (HDLs). Overweight individuals with metabolic syndrome can prevent or delay the onset of diabetes through a program of weight loss. Exercise is also important but normal weight is most important.

A patient is hospitalized after a successful resuscitation of an episode of sudden cardiac death (SCD). During the care of the patient, what nursing intervention is most important? a. Continuous ECG monitoring b. Auscultation of the carotid arteries c. Frequent assessment of heart sounds d. Monitoring of airway status and respiratory patterns

a. Most patients who experience sudden cardiac death (SCD) as a result of CAD do not have an acute MI but have dysrhythmias that cause death, probably as a result of electrical instability of the myocardium. To identify and treat those specific dysrhythmias, continuous monitoring is important. The other assessments can be done but are not the most important after an episode of SCD.

Before selecting a weight reduction plan with an obese patient, what is most important for the nurse to first assess? a. The patient's motivation to lose weight b. The length of time that the patient has been obese c. Whether financial considerations will affect the patient's choices d. The patient's anthropometric measures of height, weight, BMI, waist-to-hip ratio, and skinfold thickness

a. Motivation is essential. Focus on the reasons for wanting to lose weight. The rest of the options will assist in planning the weight loss if the patient is motivated.

To prepare a woman who has been raped for physical exam, what should the nurse do first? a. Ensure that a signed informed consent is obtained from the patient. b. Provide a private place for the patient to talk about what happened to her. c. Instruct the patient not to wash, eat, drink, or urinate before the examination. d. Administer prophylaxis for sexually transmitted infections (STIs) and tetanus.

a. Specific informed consent must be obtained from the rape victim before any examination can be made or rape data collected. Following consent, the patient is advised not to wash, eat, drink, or urinate before the examination so that evidence can be collected for medicolegal use. Prophylaxis for STIs, hepatitis B, and tetanus is administered following examination and follow-up testing for pregnancy and human immunodeficiency virus (HIV) is done in several weeks.

The patient was in a traffic collision and is experiencing loss of function below C4. Which effect must the nurse be aware of to provide priority care for the patient? a. Respiratory diaphragmatic breathing b. Loss of all respiratory muscle function c. Decreased response of the sympathetic nervous system d. GI hypomotility with paralytic ileus and gastric distention

a. Spinal injury below C4 will result in diaphragmatic breathing and usually hypoventilation from decreased vital capacity and tidal volume from intercostal muscle impairment. The nurse's priority actions will be to monitor rate, rhythm, depth, and effort of breathing to observe for changes from the baseline and identify the need for ventilation assistance. Loss of all respiratory muscle function occurs above C4 and the patient requires mechanical ventilation to survive. Although the decreased sympathetic nervous system response (from injuries above T6) and GI hypomotility (paralytic ileus and gastric distention) will occur (with injuries above T5), they are not the patient's initial priority needs.

The husband and daughter of a Hispanic woman dying from pancreatic cancer refuse to consider using hospice care. What is the first thing the nurse should do? a. Assess their understanding of what hospice care services are. b. Ask them how they will care for the patient without hospice care. c. Talk directly to the patient and family to see if she can change their minds. d. Accept their decision since they are Hispanic and prefer to care for their own.

a. The family may not understand what hospice care is and may need information. Some cultures and ethnic groups may underuse hospice care because of a lack of awareness of the services offered, a desire to continue with potentially curative therapies, and concerns about a lack of minority hospice workers.

A patient with a history of alcohol abuse is admitted to the hospital following an automobile accident. What is most important for the nurse to assess to plan care for the patient? a. When the patient last had alcohol intake b. How much alcohol has recently been used c. What type of alcohol has recently been ingested d. The patient's current blood alcohol concentration

a. The knowledge of when the patient last had alcohol intake will help the nurse to anticipate the onset of withdrawal symptoms. In patients with alcohol tolerance, the amount of alcohol and the blood alcohol concentration do not reflect impairment as consistently as in the nondrinker. The type of alcohol ingested is not important because in the body it is all alcohol.

A patient is admitted to the emergency department with acute abdominal pain. What nursing intervention should the nurse implement first? a. Measurement of vital signs b. Administration of prescribed analgesics c. Assessment of the onset, location, intensity, duration, and character of the pain d. Physical assessment of the abdomen for distention, bowel sounds, and pigmentation changes

a. The patient with an acute abdomen may have significant fluid or blood loss into the abdomen and evaluation of blood pressure (BP) and heart rate (HR) should be the first intervention, followed by assessment of the abdomen and the nature of the pain. Analgesics should be used cautiously until a diagnosis can be determined so that symptoms are not masked.

What is most important for the nurse to teach the female patient with genital warts? a. Have an annual Papanicolaou (Pap) test. b. Apply topical acyclovir faithfully as directed. c. Have her sexual partner treated for the condition. d. Use a contraceptive to prevent pregnancy, which might exacerbate the disease.

a. There is a strong association of genital warts with the development of dysplasia and neoplasia of the genital tract, especially when lesions involve the cervix, introitus, and perianal and intraanal mucosa of women or the penis and perianal and anal mucosa of men. Regular Papanicolaou (Pap) tests in women are critical in detecting early malignancies of the cervix. Oral acyclovir is used to treat HSV-2 but topical use has no value in treating viral STIs. Sexual partners of patients with HPV should be examined and treated but because treatment does not destroy the virus, condoms should always be used during sexual activity. Genital warts often grow more rapidly during pregnancy but pregnancy is not contraindicated.

A patient on the cardiac telemetry unit goes into ventricular fibrillation and is unresponsive. Following initiation of the emergency call system (Code Blue), what is the next priority for the nurse in caring for this patient? a. Begin CPR. b. Get the crash cart. c. Administer amiodarone IV. d. Defibrillate with 360 joules.

a. Until the defibrillator is available, the patient needs CPR. Defibrillation is needed as soon as possible, so someone should bring the crash cart to the room. Defibrillation would be with 360 joules for monophasic defibrillators and 120 to 200 joules for biphasic defibrillators. Amiodarone is an antidysrhythmic that is part of the advanced cardiac life support (ACLS) protocol for ventricular fibrillation.

A patient treated for vomiting is to begin oral intake when the symptoms have subsided. To promote rehydration, the nurse plans to administer which fluid first? a. Water b. Hot tea c. Gatorade d. Warm broth

a. Water is the fluid of choice for rehydration by mouth. Very hot or cold liquids are not usually well tolerated and although broth and Gatorade have been used for the patient with severe vomiting, these substances are high in sodium and should be administered with caution.

A patient diagnosed with class 3 TB 1 week ago is admitted to the hospital with symptoms of chest pain and coughing. What nursing action has the highest priority? a. Administering the patient's antitubercular drugs b. Admitting the patient to an airborne infection isolation room c. Preparing the patient's room with suction equipment and extra linens d. Placing the patient in an intensive care unit where he can be closely monitored

b. A patient with class 3 TB has clinically active disease and airborne infection isolation is required for active disease until the patient is noninfectious, indicated by negative sputum smears. Cardiac monitoring and observation will need to be done with the patient in isolation. The nurse will administer the antitubercular drugs after the patient is in isolation. There should be no need for suction or extra linens after the TB patient is receiving drug therapy.

A patient is admitted to the emergency department with profuse bright-red hematemesis. During the initial care of the patient, what is the nurse's first priority? a. Establish two IV sites with large-gauge catheters b. Perform a focused nursing assessment of the patient's status c. Obtain a thorough health history to assist in determining the cause of the bleeding d. Perform a gastric lavage with cool tap water in preparation for endoscopic examination

b. Although all of the interventions may be indicated when a patient has upper GI bleeding, the first nursing priority with bright-red (arterial) blood is to perform a focused assessment of the patient's condition, with emphasis on blood pressure (BP), pulse, and peripheral perfusion to determine the presence of hypovolemic shock.

To reduce the risk for most occupational lung diseases, what is the most important measure the occupational nurse should promote? a. Maintaining smoke-free work environments for all employees. b. Using masks and effective ventilation systems to reduce exposure to irritants. c. Inspection and monitoring of workplaces by national occupational safety agencies. d. Requiring periodic chest x-rays and pulmonary function tests for exposed employees.

b. Although all of the precautions identified in this question are appropriate in decreasing the risk of occupational lung diseases, using masks and effective ventilation systems to reduce exposure is the most efficient and affects the greatest number of employees.

When caring for a patient following a glossectomy with dissection of the floor of the mouth and a radical neck dissection for cancer of the tongue, what is the nurse's primary concern? a. Relief of pain b. Patent airway c. Positive body image d. Tube feedings to provide nutrition

b. Because surgical treatment of oral cancers involves extensive excision, a tracheostomy is usually performed with the radical dissections. The first goal of care is that the patient will have a patent airway. The other goals are appropriate but of lesser priority.

When caring for a patient with irritable bowel syndrome (IBS), what is most important for the nurse to do? a. Recognize that IBS is a psychogenic illness that cannot be definitively diagnosed. b. Develop a trusting relationship with the patient to provide support and symptomatic care. c. Teach the patient that a diet high in fiber will relieve the symptoms of both diarrhea and constipation. d. Inform the patient that new medications for IBS are available and effective for treatment of IBS manifested by either diarrhea or constipation.

b. Because there is no definitive treatment for irritable bowel syndrome (IBS) and patients become frustrated and discouraged with uncontrolled symptoms, it is important to develop a trusting relationship that will support the patient as different treatments are implemented and evaluated. Diagnosis of IBS can be established by Rome criteria and by elimination of other problems. Although IBS can be precipitated and aggravated by stress and emotions, it is not a psychogenic illness. Highfiber diets may help but they might also increase the bloating and gas pains of IBS. Medications are available but usually used as a last resort because of side effects.

Following a renal biopsy, what is the nurse's priority? a. Offer warm sitz baths to relieve discomfort. b. Test urine for microscopic bleeding with a dipstick. c. Expect the patient to experience burning on urination. d. Monitor the patient for symptoms of a urinary infection.

b. Bleeding from the kidney following a biopsy is the most serious complication of the procedure and urine must be examined for both gross and microscopic blood, in addition to vital signs and hematocrit levels being monitored. Following a cystoscopy the patient may have burning with urination and warm sitz baths may be used. Urinary infections are a complication of any procedure requiring instrumentation of the bladder

Following a patient's esophagogastrostomy for cancer of the esophagus, what is most important for the nurse to do? a. Report any bloody drainage from the NG tube. b. Maintain the patient in semi-Fowler's or Fowler's position. c. Monitor for abdominal distention that may disrupt the surgical site. d. Expect to find decreased breath sounds bilaterally because of the surgical approach.

b. Following esophageal surgery, the patient should be positioned in semi-Fowler's or Fowler's position to prevent reflux and aspiration of gastric sections. NG drainage is expected to be bloody for 8 to 12 hours postoperatively. Abdominal distention is not a major concern following esophageal surgery and even though the thorax may be opened during the surgery, clear breath sounds should be expected in all areas of the lungs.

A patient with a gunshot wound to the abdomen is being treated for hypovolemic and septic shock. To monitor the patient for early organ damage associated with MODS, what is most important for the nurse to assess? a. Urine output b. Breath sounds c. Peripheral circulation d. Central venous pressure

b. Generally, the first body system affected by mediatorinduced injury in MODS is the respiratory system. Adventitious sounds and areas with absent breath sounds will be present. Other organ damage occurs but lungs are usually first.

A patient with chronic heart failure is treated with hydrochlorothiazide, digoxin, and lisinopril (Prinivil). To prevent the risk of digitalis toxicity with these drugs, what is most important that the nurse monitor for this patient? a. Heart rate (HR) b. Potassium levels c. Blood pressure (BP) d. Gastrointestinal function

b. Hypokalemia is one of the most common causes of digitalis toxicity because low serum potassium levels enhance ectopic pacemaker activity. When a patient is receiving potassium-losing diuretics, such as hydrochlorothiazide or furosemide, it is essential to monitor the patient's serum potassium levels to prevent digitalis toxicity. Monitoring the heart rate (HR) assesses for complications related to digoxin but does not prevent toxicity.

While the nurse performs range of motion (ROM) on an unconscious patient with increased ICP, the patient experiences severe decerebrate posturing reflexes. What should the nurse do first? a. Use restraints to protect the patient from injury. b. Perform the exercises less frequently because posturing can increase ICP. c. Administer central nervous system (CNS) depressants to lightly sedate the patient. d. Continue the exercises because they are necessary to maintain musculoskeletal function.

b. If reflex posturing occurs during range of motion (ROM) or positioning of the patient, these activities should be done less frequently until the patient's condition stabilizes because posturing can cause increases in ICP and may indicate herniation. Neither restraints nor central nervous system (CNS) depressants would be indicated.

A dehydrated patient is in the Injury stage of the RIFLE staging of AKI. What would the nurse first anticipate in the treatment of this patient? a. Assess daily weight b. IV administration of fluid and furosemide (Lasix) c. IV administration of insulin and sodium bicarbonate d. Urinalysis to check for sediment, osmolality, sodium, and specific gravity

b. Injury is the stage of RIFLE classification when urine output is less than 0.5 mL/kg/hr for 12 hours, the serum creatinine is increased times two or the glomerular filtration rate (GFR) is decreased by 50%. This stage may be reversible by treating the cause or, in this patient, the dehydration by administering IV fluid and a low dose of a loop diuretic, furosemide (Lasix). Assessing the daily weight will be done to monitor fluid changes but it is not the first treatment the nurse should anticipate. IV administration of insulin and sodium bicarbonate would be used for hyperkalemia. Checking the urinalysis will help to determine if the AKI has a prerenal, intrarenal, or postrenal cause by what is seen in the urine but with this patient's dehydration, it is thought to be prerenal to begin treatment.

A patient with severe ulnar deviation of the hands undergoes an arthroplasty with reconstruction and replacement of finger joints. Postoperatively, what is it most important for the nurse to do? a. Position the fingers lower than the elbow. b. Perform neurovascular assessments of the fingers q2-4hr. c. Encourage the patient to gently flex, extend, abduct, and adduct the fingers q4hr. d. Remind the patient that function of the hands is more important than their cosmetic appearance.

b. Neurovascular checks of the fingers following surgery of the hands are essential to detect compromised vascular and neurologic function caused by trauma or edema. Postoperatively, the hands are elevated with a bulky dressing in place and when the dressing is removed, a guided splinting program is started. Exercises are performed three to four times a day when the splints are removed and the patient is discharged. Before surgery, it must be made clear to the patient that the goal of the surgery is to restore function related to grasp, pinch, stability, and strength and the hands will not necessarily have good cosmetic appearance.

The nurse admitting a patient for bariatric surgery obtains the following information from the patient. Which finding should be brought to the surgeon's attention before proceeding with further patient preparation? a. History of hypertension b. History of untreated depression c. History of multiple attempts at weight loss d. History of sleep apnea treated with continuous positive airway pressure (CPAP)

b. Patients with histories of untreated depression or psychosis are not good candidates for surgery. All other historical information includes medical complications of severe obesity that would help to qualify the patient for the surgery.

On admission of a victim of sexual assault to the emergency department, what should be the first priority of the nurse? a. Contact a rape support person for the patient. b. Assess the patient for urgent medical problems. c. Question the patient about the details of the assault. d. Inform the patient what procedures and treatments will be performed.

b. Sexual assault is an act of violence and the first priority of care for the patient should be assessment and treatment of serious injuries involving extragenital areas, such as fractures, subdural hematomas, cerebral concussions, and intraabdominal injuries. All of the other options are appropriate treatments but treatment for shock and urgent medical injuries is the first priority.

The nurse caring for a patient immediately following a transesophageal echocardiogram (TEE) should consider which action the highest priority? a. Monitor the ECG b. Monitor pulse oximetry c. Assess vital signs (BP, HR, RR, temperature) d. Maintain NPO status until gag reflex has returned

d. All actions will be done but in order to perform a transesophageal echocardiogram (TEE), the throat must be numbed. Until sensation returns, as evidenced by the gag reflex, the patient is at risk of aspiration so this action has the highest priority (priority related to airway—ABCs).

A patient is receiving chemotherapy. She calls the physician's office and says she is experiencing itching in her groin and under her breasts. What is the first nursing assessment that would be done before the nurse makes an appointment for the patient with the physician to determine the treatment? a. Her height and weight b. What the areas look like c. If chemotherapy was completed d. Culture and sensitivity of the areas

b. The appearance of candidiasis on the skin shows diffuse papular erythematous rash with pinpoint satellites around the affected area. Height and weight could show if the patient is obese but it would be better to ask if the areas affected are moist. The chemotherapy could contribute to candidiasis but it does not matter if the chemotherapy treatments are finished. Culture and sensitivity of the area may be ordered by the physician at the patient's appointment.

A patient has an oral ET tube inserted to relieve an upper airway obstruction and to facilitate secretion removal. What is the first responsibility of the nurse immediately following placement of the tube? a. Suction the tube to remove secretions. b. Place an end tidal CO2 detector on the ET tube. c. Secure the tube to the face with adhesive tape. d. Assess for bilateral breath sounds and symmetric chest movement.

b. The first action of the nurse is to use an end tidal CO2 detector. If no CO2 is detected, the tube is in the esophagus. The second action by the nurse following ET intubation is to auscultate the chest to confirm bilateral breath sounds and observe to confirm bilateral chest expansion. If this evidence is present, the tube is secured and connected to an O2 source. Then the placement is confirmed immediately with x-ray and the tube is marked where it exits the mouth. The patient should be suctioned as needed.

A patient with continuous bladder irrigation following a prostatectomy tells the nurse that he has bladder spasms and leaking of urine around the catheter. What should the nurse do first? a. Slow the rate of the irrigation. b. Assess the patency of the catheter. c. Encourage the patient to try to urinate around the catheter. d. Administer a belladonna and opium (B&O) suppository as prescribed.

b. The nurse should first check for the presence of clots obstructing the catheter or tubing and then may administer a belladonna and opium (B&O) suppository if one is ordered. The patient should not try to void around the catheter because this will increase the spasms. The flow rate of the irrigation fluid may be decreased if orders permit because fast-flowing, cold fluid may also contribute to spasms.

To decrease the patient's sense of panic during an acute asthma attack, what is the best action for the nurse to do? a. Leave the patient alone to rest in a quiet, calm environment. b. Stay with the patient and encourage slow, pursed lip breathing. c. Reassure the patient that the attack can be controlled with treatment. d. Let the patient know that frequent monitoring is being done using measurement of vital signs and SpO2.

b. The patient in an acute asthma attack is very anxious and fearful. It is important to stay with the patient and interact in a calm, unhurried manner. Helping the patient to breathe with pursed lips will facilitate expiration of trapped air and help the patient to gain control of breathing. Pursed lip breathing also is used with COPD for this same reason.

A 46-year-old African American patient is scheduled to have a basal cell carcinoma on his cheek excised in the health care provider's office. What factor is most important for the nurse to obtain in the patient's history? a. Protected sun exposure b. Radiation treatment for acne c. Prior treatments for the lesion d. Exposure to harsh irritants such as ammonia

b. Thirty years ago, when the patient was a teenager, radiation therapy was used to treat cystic acne with the result that many of these patients now have developed basal cell carcinoma. For a person with dark skin, radiation therapy is a higher risk factor for skin cancer than exposure to the sun or other irritants.

While the nurse is obtaining a health history for a patient, the patient tells the nurse that he uses a number of herbs to maintain his health. What is the most important thing the nurse can do to address the patient's use of these products? a. Ask the patient what effects the various products have. b. Have a working knowledge of commonly used herbs and dietary supplements. c. Reassure the patient that the products can continue to be used with conventional therapies. d. Warn the patient that there is limited research on the therapeutic and harmful effects of herbal products.

b. To serve as a resource for patients regarding complementary and alternative therapies, nurses must first develop their own knowledge base. To teach patients about their use, monitor for adverse effects and interactions with conventional therapies, and assist patients to use the therapies knowledgeably and safely, nurses themselves must be informed.

Before administering a bolus of intermittent tube feeding to a patient with a percutaneous endoscopic gastrostomy (PEG), the nurse aspirates 220 mL of gastric contents. How should the nurse respond? a. Return the aspirate to the stomach and recheck the volume of aspirate in an hour. b. Return the aspirate to the stomach and continue with the tube feeding as planned. c. Discard the aspirate to prevent overdistending the stomach when the new feeding is given. d. Notify the health care provider that the feedings have been scheduled too frequently to allow for stomach emptying.

b. With intermittent feedings, less than 250 mL residual does not require further action. With continuous feedings and a residual of 250 mL or more after the second residual check, a promotility agent should be considered.

When caring for a patient following a radical prostatectomy with a perineal approach, what is the priority nursing intervention the nurse should use to prevent complications? a. Use chemotherapeutic agents to prevent metastasis. b. Administer sildenafil (Viagra) as needed for erectile dysfunction. c. Provide wound care after each bowel movement to prevent infection. d. Insert a smaller indwelling urinary catheter to prevent urinary retention.

c. A prostatectomy performed with a perineal approach has a high risk for infection because of the proximity of the wound to the anus, so wound care is the priority. Chemotherapy is usually not the first choice of drug therapy following surgery, nor is sildenafil. The catheter size would not be changed but the catheter would be removed. Urinary incontinence is a bigger problem than retention.

When replacement therapy is started for a patient with long-standing hypothyroidism, what is most important for the nurse to monitor the patient for? a. Insomnia b. Weight loss c. Nervousness d. Dysrhythmias

d. All these manifestations may occur with treatment of hypothyroidism. However, as a result of the effects of hypothyroidism on the cardiovascular system, when thyroid replacement therapy is started myocardial oxygen consumption is increased and the resultant oxygen demand may cause angina, cardiac dysrhythmias, and heart failure, so monitoring for dysrhythmias is most important.

After the health care provider sees a patient hospitalized with a stroke who developed a fever and adventitious lung sounds, the following orders are written. Which order should the nurse implement first? a. Anterior/posterior and lateral chest x-rays b. Start IV levofloxacin (Levaquin) 500 mg every 24 hr c. Sputum specimen for Gram stain and culture and sensitivity d. Complete blood count (CBC) with white blood cell (WBC) count and differential

c. A sputum specimen for Gram stain and culture should be done before initiating antibiotic therapy and while waiting for the antibiotic to be delivered from the pharmacy in a hospitalized patient with suspected pneumonia and then antibiotics should be started without delay. If the sputum specimen cannot be obtained rapidly, the chest x-ray will be done to assess the typical pattern characteristic of the infecting organism. Blood cell tests will not be altered significantly by delaying the tests until after the first dose of antibiotics.

Before repositioning the patient on the side after a lumbar laminectomy, what should be the nurse's first action? a. Raise the head of the bed 30 degrees. b. Have the patient flex the knees and hips. c. Place a pillow between the patient's legs. d. Have the patient grasp the side rail on the opposite side of the bed.

c. After spinal surgery, patients are logrolled to maintain straight alignment of the spine at all times, requiring the patient to be turned with a pillow between the legs and moving the body as a unit. The head of the bed is usually kept flat and the legs are extended.

A patient returns to the surgical unit with a nasogastric (NG) tube to low intermittent suction, IV fluids, and a Jackson-Pratt drain at the surgical site following an exploratory laparotomy and repair of a bowel perforation. Four hours after admission, the patient experiences nausea and vomiting. What is a priority nursing intervention for the patient? a. Assess the abdomen for distention and bowel sounds. b. Inspect the surgical site and drainage in the Jackson-Pratt. c. Check the amount and character of gastric drainage and the patency of the NG tube. d. Administer prescribed prochlorperazine (Compazine) to control the nausea and vomiting.

c. An adequately functioning nasogastric (NG) tube should prevent nausea and vomiting because stomach contents are continuously being removed. The first intervention in this case is to check the amount and character of the recent drainage and check the tube for patency. Decreased or absent bowel sounds are expected after a laparotomy and the Jackson-Pratt drains only fluid from the tissue of the surgical site. Antiemetics may be given if the NG tube is patent because anesthetic agents may cause nausea.

A patient in alcohol withdrawal has a nursing diagnosis of ineffective protection related to sensorimotor deficits, seizure activity, and confusion. Which nursing intervention is most important for the patient? a. Provide a darkened, quiet environment free from external stimuli. b. Force fluids to assist in diluting the alcohol concentration in the blood. c. Monitor vital signs frequently to detect an extreme autonomic nervous system response. d. Use restraints as necessary to prevent the patient from reacting violently to hallucinations.

c. An extreme autonomic nervous system response may be life threatening and requires immediate intervention. A quiet room is recommended but it should be well lighted to prevent misinterpretation of the environment and visual hallucinations. Cessation of alcohol intake causes low blood alcohol levels leading to withdrawal symptoms and fluids should be administered carefully to prevent dysrhythmias.

During assessment of a patient with a spinal cord injury, the nurse determines that the patient has a poor cough with diaphragmatic breathing. Based on this finding, what should be the nurse's first action? a. Institute frequent turning and repositioning. b. Use tracheal suctioning to remove secretions. c. Assess lung sounds and respiratory rate and depth. d. Prepare the patient for endotracheal intubation and mechanical ventilation.

c. Because pneumonia and atelectasis are potential problems related to ineffective coughing and the loss of intercostal and abdominal muscle function, the nurse should assess the patient's breath sounds and respiratory function to determine whether secretions are being retained or whether there is progression of respiratory impairment. Suctioning is not indicated unless lung sounds indicate retained secretions. Position changes will help to mobilize secretions. Intubation and mechanical ventilation are used if the patient becomes exhausted from labored breathing or if arterial blood gases (ABGs) deteriorate.

During the treatment of the patient with bleeding esophageal varices, what is the most important thing the nurse should do? a. Prepare the patient for immediate portal shunting surgery. b. Perform guaiac testing on all stools to detect occult blood. c. Maintain the patient's airway and prevent aspiration of blood. d. Monitor for the cardiac effects of IV vasopressin and nitroglycerin.

c. Bleeding esophageal varices are a medical emergency. During an episode of bleeding, management of the airway and prevention of aspiration of blood are critical factors. Occult blood as well as fresh blood from the GI tract would be expected. Vasopressin causes vasoconstriction, decreased heart rate, and decreased coronary blood flow. IV nitroglycerin may be given with the vasopressin to counter these side effects. Portal shunting surgery is performed for esophageal varices but not during an acute hemorrhage.

Following an ascending aortic aneurysm repair, what is an important finding that the nurse should report immediately to the health care provider? a. Shallow respirations and poor coughing b. Decreased drainage from the chest tubes c. A change in level of consciousness (LOC) and inability to speak d. Lower extremity pulses that are decreased from the preoperative baseline

c. During repair of an AAA, the blood supply to the carotid arteries may be interrupted, leading to neurologic complications manifested by a decreased level of consciousness (LOC) and altered pupil responses to light as well as changes in facial symmetry, speech, and movement of the upper extremities. The thorax is opened for ascending aortic surgery and shallow breathing, poor cough, and decreasing chest drainage are expected. Often, lower limb pulses are normally decreased or absent for a short time following surgery.

The nurse receives an evening report on a patient who underwent posterior nasal packing for epistaxis earlier in the day. What is the first patient assessment the nurse should make? a. Patient's temperature b. Level of the patient's pain c. Drainage on the nasal dressing d. Oxygen saturation by pulse oximetry

d. All of the assessments are appropriate but the most important is the patient's oxygen status. After the posterior nasopharynx is packed, some patients, especially older adults, experience a decrease in PaO2 and an increase in PaCO2 because of impaired respiration and the nurse should monitor the patient's respiratory rate and rhythm and SpO2.

A patient with esophageal cancer is scheduled for a partial esophagectomy. Which nursing intervention is likely to be of highest priority preoperatively? a. Practice turning and deep breathing. b. Brush the teeth and mouth well each day. c. Encourage a high-calorie, high-protein diet. d. Teach about postoperative tubes and cares.

c. Eating a high-calorie, high-protein diet, perhaps in liquid form, is the highest priority preoperatively. Because of dysphagia, the patient frequently has poor nutritional status because of the inability to ingest adequate amounts of food before surgery. An esophageal stent may be placed to improve the nutritional status. Turning and deep breathing will be done and the patient will need to know about postoperative care but these are not the priorities. Meticulous oral care is done but it may be done with swabs or gauze pads to prevent the injury and pain brushing may incur.

When teaching a patient with a seizure disorder about the medication regimen, what is it most important for the nurse to emphasize? a. The patient should increase the dosage of the medication if stress is increased. b. Most over-the-counter and prescription drugs are safe to take with antiseizure drugs. c. Stopping the medication abruptly may increase the intensity and frequency of seizures. d. If gingival hypertrophy occurs, the drug should be stopped and the health care provider notified.

c. If antiseizure drugs are discontinued abruptly, seizures can be precipitated. Missed doses should be made up if the omission is remembered within 24 hours and patients should not adjust medications without professional guidance because this also can increase seizure frequency and may cause status epilepticus. Antiseizure drugs have numerous interactions with other drugs and the use of other medications should be evaluated by health professionals. If side effects occur, the physician should be notified and drug regimens evaluated.

Older patients may have cardiac or renal insufficiency and may be more susceptible to problems from vomiting and antiemetic drug side effects. What nursing intervention is most important to implement with these patients? a. Keep the patient flat in bed to decrease dizziness. b. Keep the patient NPO until nausea and vomiting has stopped. c. Do hourly visual checks or use a sitter to keep the patient safe. d. Administer IV fluids as rapidly as possible to prevent dehydration.

c. Implementing safety precautions (placement close to the nurses' station, call bell in reach, hourly visual checks, use of sitters) is the priority. The patient would not be kept in a flat position because of the potential for aspiration of vomitus. Because the older patient is more likely to have cardiac or renal insufficiency, the patient's fluid and electrolyte status are monitored more closely (laboratory, intake and output). Monitor vital signs along with breath sounds. Assess mucous membranes, skin turgor, and color to assess for dehydration. Assess level of consciousness closely. Check dosing of antiemetics. Assess for weakness and fatigue.

A 76-year-old woman has an Hgb of 7.3 g/dL (73 g/L) and is experiencing ataxia and confusion on admission to the hospital. What is a priority nursing intervention for this patient? a. Provide a darkened, quiet room. b. Have the family stay with the patient. c. Keep top bedside rails up and call bell in close reach d. Question the patient about possible causes of anemia

c. In the older adult, confusion, ataxia, and fatigue are common manifestations of anemia and place the patient at risk for injury. Nursing interventions should include safety precautions to prevent falls and injury when these symptoms are present. The nurse, not the patient's family, is responsible for the patient and although a quiet room may promote rest, it is not as important as protection of the patient.

To promote effective coughing, deep breathing, and ambulation in the postoperative patient, what is most important for the nurse to do? a. Teach the patient controlled breathing b. Explain the rationale for these activities c. Provide adequate and regular pain medication d. Use an incentive spirometer to motivate the patient

c. Incisional pain is often the greatest deterrent to patient participation in effective ventilation and ambulation and adequate and regular analgesic medications should be provided to encourage these activities. Controlled breathing may help the patient to manage pain but does not promote coughing and deep breathing. Explanations and use of an incentive spirometer help to gain patient participation but are more effective if pain is controlled.

During care of the patient following femoral bypass graft surgery, the nurse immediately notifies the health care provider if the patient experiences a. fever and redness at the incision site. b. 2+ edema of the extremity and pain at the incision site. c. a loss of palpable pulses and numbness and tingling of the feet. d. increasing ankle-brachial indices and serous drainage from the incision.

c. Loss of palpable pulses, numbness and tingling of the extremity, extremity pallor, cyanosis or cold, and decreasing ankle-brachial indices are indications of occlusion of the bypass graft and need immediate medical attention. Pain, redness, and serous drainage at the incision site are expected postoperatively.

A bedridden patient tells the nurse she has low back pain and asks if the area could be massaged. What is the best action by the nurse? a. Ask the patient if she has ever tried acupuncture for back pain. b. Explain to the patient that massage may be done only by a licensed therapist and offer a PRN analgesic instead. c. Comfortably position the patient to expose the area and massage the back with effleurage and petrissage strokes. d. Call the physical therapy department to request that a physical therapist see the patient to provide a therapeutic massage.

c. Massage therapy is one of the body-based methods commonly used by nurses for many different effects. As part of nursing care, nurses can use specific massage techniques that are not as comprehensive as those of massage therapists. It would be quite appropriate for the nurse to massage the patient's back. The other options are inappropriate at this time.

A patient with an acute MI is having multifocal PVCs and ventricular couplets. He is alert and has a BP of 118/78 mm Hg with an irregular pulse of 86 bpm. What is the priority nursing action at this time? a. Continue to assess the patient. b. Ask the patient to perform Valsalva maneuver. c. Prepare to administer antidysrhythmic drugs per protocol. d. Be prepared to administer cardiopulmonary resuscitation (CPR).

c. Multifocal PVCs in a patient with an MI indicate significant ventricular irritability that may lead to ventricular tachycardia or ventricular fibrillation. Antidysrhythmics, such as β-adrenergic blockers, procainamide, amiodarone, or lidocaine, may be used to control the dysrhythmias. Valsalva maneuver may be used to treat paroxysmal supraventricular tachycardia. The nurse must always be ready to perform cardiopulmonary resuscitation (CPR).

A patient is admitted to the emergency department with acute decompensated heart failure (ADHF). Which IV medication would the nurse expect to administer first? a. Digoxin (Lanoxin) b. Morphine sulfate c. Nesiritide (Natrecor) d. Bumetanide (Bumex)

c. Nesiritide (Natrecor) is a recombinant form of a natriuretic peptide that decreases preload and afterload by reducing pulmonary artery wedge pressure (PAWP) and systolic BP which decreases the workload of the heart for short-term emergency treatment of acute decompensated heart failure (ADHF). Digoxin (Lanoxin) requires a loading dose and time to work, so it is not recommended for emergency treatment of ADHF. Morphine sulfate relieves dyspnea but has more adverse events and mortality. Bumetanide (Bumex) will decrease fluid volume but also will decrease potassium levels and activate the sympathetic nervous system and renin-angiotensin-aldosterone system, which can exacerbate HF symptoms.

What is the priority nursing responsibility in the prevention of shock? a. Frequently monitoring all patients' vital signs b. Using aseptic technique for all invasive procedures c. Being aware of the potential for shock in all patients at risk d. Teaching patients health promotion activities to prevent shock

c. Prevention of shock necessitates identification of persons who are at risk and a thorough baseline nursing assessment with frequent ongoing assessments to monitor and detect changes in patients at risk. Frequent monitoring of all patients' vital signs is not necessary. Aseptic technique for all invasive procedures should always be implemented but will not prevent all types of shock. Health promotion activities that reduce the risk for precipitating conditions, such as coronary artery disease or anaphylaxis, may help to prevent shock in some cases.

A pulse oximetry monitor indicates that the patient has a drop in SpO2 from 95% to 85% over several hours. What is the first action the nurse should take? a. Order stat ABGs to confirm the SpO2 with a SaO2. b. Start oxygen administration by nasal cannula at 2 L/min. c. Check the position of the probe on the finger or earlobe. d. Notify the health care provider of the change in baseline PaO2.

c. Pulse oximetry is inaccurate if the probe is loose, if there is low perfusion, or when skin color is dark. Before other measures are taken, the nurse should check the probe site. If the probe is intact at the site and perfusion is adequate, an ABG analysis will be ordered by the health care provider to verify accuracy and oxygen may be administered, depending on the patient's condition and the assessment of respiratory and cardiac status.

The nurse is assessing a newly admitted diabetic patient. Which observation should be addressed as the priority by the nurse? a. Bilateral numbness of both hands b. Stage II pressure ulcer on the right heel c. Rapid respirations with deep inspiration d. Areas of lumps and dents on the abdomen

c. Rapid deep respirations are symptoms of diabetic ketoacidosis (DKA), so this is the priority of care. Stage II pressure ulcers and bilateral numbness are chronic complications of diabetes. The lumps and dents on the abdomen indicate a need to teach the patient about site rotation.

What is the most important nursing intervention for the prevention and treatment of pressure ulcers? a. Using pressure-reduction devices b. Massaging pressure areas with lotion c. Repositioning the patient a minimum of every 2 hours d. Using lift sheets and trapeze bars to facilitate patient movement

c. Relief of pressure on tissues is critical to prevention and treatment of pressure ulcers. Although pressurereduction devices may relieve some pressure and lift sheets and trapeze bars prevent skin shear, they are no substitute for frequent repositioning of the patient. Massage is contraindicated if there is the presence of acute inflammation or possibly damaged blood vessels or fragile skin.

When the nurse asks a preoperative patient about allergies, the patient reports a history of seasonal environmental allergies and allergies to a variety of fruits. What should the nurse do next? a. Note this information in the patient's record as hay fever and food allergies. b. Place an allergy alert wristband that identifies the specific allergies on the patient. c. Ask the patient to describe the nature and severity of any allergic responses experienced from these agents. d. Notify the anesthesia care provider (ACP) because the patient may have an increased risk for allergies to anesthetics.

c. Risk factors for latex allergies include a history of hay fever and allergies to foods such as avocados, kiwi, bananas, potatoes, peaches, and apricots. When a patient identifies such allergies, the patient should be further questioned about exposure to latex and specific reactions to allergens. A history of any allergic responsiveness increases the risk for hypersensitivity reactions to drugs used during anesthesia but the hay fever and fruit allergies are specifically related to latex allergy. After identifying the allergic reaction, the anesthesia care provider (ACP) should be notified, the allergy alert wristband should be applied, and the note in the record will include the allergies and reactions as well as the nursing actions related to the allergies.

During care of the severely obese patient, what is most important for the nurse to do? a. Avoid reference to the patient's weight to avoid embarrassing the patient. b. Emphasize to the patient how important it is to lose weight to maintain health. c. Plan for necessary modifications in equipment and nursing techniques before initiating care. d. Recognize that a full assessment of each body system might not be possible because of numerous layers of skinfolds.

c. Special considerations are needed for the care of the severely obese patient because most hospital units are not prepared with beds, chairs, BP cuffs, and other equipment that will need to be used with the very obese patient. Consideration of all aspects of care should be made before implementing care for the patient, including extra time and perhaps assistance for positioning, physical assessment, and transferring the patient.

A patient with MS has a nursing diagnosis of self-care deficit related to muscle spasticity and neuromuscular deficits. In providing care for the patient, what is most important for the nurse to do? a. Teach the family members how to care adequately for the patient's needs. b. Encourage the patient to maintain social interactions to prevent social isolation. c. Promote the use of assistive devices so the patient can participate in self-care activities. d. Perform all activities of daily living (ADLs) for the patient to conserve the patient's energy.

c. The main goal in care of the patient with MS is to keep the patient active and maximally functional and promote self-care as much as possible to maintain independence. Assistive devices encourage independence while preserving the patient's energy. No care activity that the patient can do for himself or herself should be performed by others. Involvement of the family in the patient's care and maintenance of social interactions are also important but are not the priority in care.

A patient is admitted to the hospital for evaluation and treatment of thrombocytopenia. Which action is most important for the nurse to implement? a. Taking the temperature every 4 hours to assess for fever b. Maintaining the patient on strict bed rest to prevent injury c. Monitoring the patient for headaches, vertigo, or confusion d. Removing the oral crusting and scabs with a soft brush four times a day

c. The major complication of thrombocytopenia is hemorrhage and it may occur in any area of the body. Cerebral hemorrhage may be fatal and evaluation of mental status for central nervous system (CNS) alterations to identify CNS bleeding is very important. Fever is not a common finding in thrombocytopenia. Protection from injury to prevent bleeding is an important nursing intervention but strict bed rest is not indicated. Oral care is performed very gently with minimum friction and soft swabs.

Key interventions for treating soft tissue injury and resulting inflammation are remembered using the acronym RICE. What are the most important actions for the emergency department nurse to do for the patient with an ankle injury? a. Reduce swelling, shine light on wound, control mobility, and elicit the history of the injury b. Rub the wound clean, immobilize the area, cover the area protectively, and exercise that leg c. Rest with immobility, apply a cold compress, apply a compress bandage, and elevate the ankle d. Rinse the wounded ankle, image the ankle, carry the patient, and extend the ankle with imaging

c. The nurse will encourage Rest to prevent further injury. Ice or cold compresses will be applied to decrease swelling with vasoconstriction. Compression will help to reduce edema and stop bleeding if it is occurring. Elevation will help to decrease edema and pain. The other options are not correct.

A patient taking insulin has recorded fasting glucose levels above 200 mg/dL (11.1 mmol/L) on awakening for the last five mornings. What should the nurse advise the patient to do first? a. Increase the evening insulin dose to prevent the dawn phenomenon. b. Use a single-dose insulin regimen with an intermediate-acting insulin. c. Monitor the glucose level at bedtime, between 2:00 am and 4:00 am, and on arising. d. Decrease the evening insulin dosage to prevent night hypoglycemia and the Somogyi effect.

c. The patient's elevated glucose on arising may be the result of either dawn phenomenon or Somogyi effect. The best way to determine whether the patient needs more or less insulin is by monitoring the glucose at bedtime, between 2:00 am and 4:00 am, and on arising. If predawn levels are below 60 mg/dL, the insulin dose should be reduced. If the 2:00 am to 4:00 am blood glucose is high, the insulin should be increased.

When caring for a patient who has had most of the stomach surgically removed, what is important for the nurse to teach the patient? a. Extra iron will need to be taken to prevent anemia. b. Avoid foods with lactose to prevent bloating and diarrhea. c. Lifelong supplementation of cobalamin (vitamin B12) will be needed. d. Because of the absence of digestive enzymes, protein malnutrition is likely

c. The stomach secretes intrinsic factor, necessary for cobalamin (vitamin B12) absorption in the intestine. When part or all of the stomach is removed, cobalamin must be supplemented for life. The other options will not be a problem.

What is the most important thing the nurse should do when caring for a patient who has contracted Clostridium difficile? a. Clean the entire room with ammonia. b. Feed the patient yogurt with probiotics. c. Wear gloves and wash hands with soap and water. d. Teach the family to use alcohol-based hand cleaners.

c. Wearing gloves will avoid hand contamination and washing hands with soap and water will remove more Clostridium difficile spores than alcohol-based hand cleaners and ammonia-based disinfectants. The entire room will need to be disinfected with a 10% solution of household bleach. Probiotics may help to prevent diarrhea in the patient on antibiotics by replacing normal intestinal bacteria.

While the nurse is obtaining a health history the patient tells the nurse, "I am so tired I can hardly function." What is the nurse's best action at this time? a. Stop the interview and leave the patient alone to be able to rest. b. Arrange another time with the patient to complete the interview. c. Question the patient further about the characteristics of the symptoms. d. Reassure the patient that the symptoms will improve when treatment has had time to be effective.

c. When a patient describes a feeling, the nurse should ask about the factors surrounding the situation to clarify the etiology of the problem. An incorrect nursing diagnosis may be made if the statement is taken literally and its meaning is not explored with the patient. A sense of "being tired and unable to function" does not necessarily indicate a need for rest or sleep and there is no way to know that treatment will relieve the problem.

What is the most appropriate snack for the nurse to offer a patient with stage 4 CKD? a. Raisins b. Ice cream c. Dill pickles d. Hard candy

d. A patient with CKD may have unlimited intake of sugars and starches (unless the patient is diabetic) and hard candy is an appropriate snack and may help to relieve the metallic and urine taste that is common in the mouth. Raisins are a high-potassium food. Ice cream contains protein and phosphate and counts as fluid. Pickled foods have high sodium content.

When caring for a patient with an acute exacerbation of a peptic ulcer, the nurse finds the patient doubled up in bed with shallow, grunting respirations. Which action should the nurse take first? a. Irrigate the patient's NG tube. b. Notify the health care provider. c. Place the patient in high-Fowler's position. d. Assess the patient's abdomen and vital signs.

d. Abdominal pain that causes the knees to be drawn up and shallow, grunting respirations in a patient with peptic ulcer disease are characteristic of perforation and the nurse should assess the patient's vital signs and abdomen before notifying the health care provider. Irrigation of the NG tube should not be performed because the additional fluid may be spilled into the peritoneal cavity and the patient should be placed in a position of comfort, usually on the side with the head slightly elevated.

A patient is admitted to the ED with nausea, vomiting, and right upper quadrant pain. The patient's family brought an empty container of acetaminophen that was found near him. A large oral gastric tube is inserted. What does the nurse prepare to administer first? a. Cathartics b. Ipecac syrup c. Gastric lavage d. Activated charcoal

d. Activated charcoal will absorb any of the medication left in the stomach. Cathartics are usually given with activated charcoal to increase elimination of the toxins absorbed by the charcoal. N-acetylcysteine (Mucomyst) will be administered for acetaminophen toxicity. Vomiting should never be induced in a patient who is unconscious.

While teaching women about the risks and incidence of CAD, what does the nurse emphasize? a. Smoking is not as significant a risk factor for CAD in women as it is in men. b. Women seek treatment sooner than men when they have symptoms of CAD. c. Estrogen replacement therapy in postmenopausal women decreases the risk for CAD. d. CAD is the leading cause of death in women, with a higher mortality rate after MI than in men.

d. CAD is the number-one killer of American women and women have a much higher mortality rate within 1 year following MI than do men. Smoking carries specific problems for women because smoking has been linked to a decrease in estrogen levels and to early menopause and it has been identified as the most powerful contributor to CAD in women under the age of 50. Fewer women than men present with classic manifestations and women delay seeking care longer than men. Recent research indicates that estrogen replacement does not reduce the risk for CAD, even though estrogen lowers low-density lipoprotein (LDL) and raises high-density lipoprotein (HDL) cholesterol.

The nurse prepares to interview a patient for a nursing history but finds the patient in obvious pain. Which action by the nurse is the best at this time? a. Delay the interview until the patient is free of pain. b. Administer pain medication before initiating the interview. c. Gather as much information as quickly as possible by using closed questions that require brief answers. d. Ask only those questions pertinent to the specific problem and complete the interview when the patient is more comfortable

d. Data are required regarding the immediate problem but gathering additional information can be delayed. The patient should not receive pain medication before pertinent information related to allergies or the nature of the problem is obtained. Questions that require brief answers do not elicit adequate information for a health profile.

After a patient had a renal arteriogram and is back on the clinical unit, what is the most important action by the nurse? a. Observe for gross bleeding in the urine. b. Place the patient in high Fowler's position. c. Monitor the patient for signs of allergy to the contrast medium. d. Assess peripheral pulses in the involved leg every 30 to 60 minutes.

d. During a renal arteriogram, a catheter is inserted, most commonly at the femoral artery. Following the procedure the patient is positioned with the affected leg extended with a pressure dressing applied. Peripheral pulse monitoring is essential to detect the development of thrombi around the insertion site, which may occlude blood supply to the leg. Gross bleeding in the urine is a complication of a renal biopsy. Allergy to the contrast medium should be established before the procedure.

While suctioning the ET tube of a spontaneously breathing patient, the nurse notes that the patient develops bradycardia with premature ventricular contractions. What should the nurse do first? a. Stop the suctioning and assess the patient for spontaneous respirations. b. Attempt to re-suction the patient with reduced suction pressure and pass time. c. Stop the suctioning and ventilate the patient with slow, small-volume breaths using a bag-valvemask (BVM) device. d. Stop suctioning and ventilate the patient with a BVM device with 100% oxygen until the HR returns to baseline.

d. If new dysrhythmias occur during suctioning, the suctioning should be stopped and the patient should be slowly ventilated via BVM with 100% oxygen until the dysrhythmia subsides. Patients with bradycardia should not be suctioned excessively. Ventilation of the patient with slow, smallvolume breaths using the BVM is performed when severe coughing results from suctioning.

While caring for a patient following a subtotal gastrectomy with a gastroduodenostomy anastomosis, the nurse determines that the NG tube is obstructed. Which action should the nurse take first? a. Replace the tube with a new one. b. Irrigate the tube until return can be aspirated. c. Reposition the tube and then attempt irrigation. d. Notify the surgeon to reposition or replace the tube.

d. If the patient's NG tube becomes obstructed following a gastrectomy with an intestinal anastomosis, gastric secretions may put a strain on the sutured anastomosis and cause serious complications. Be sure that the suction is working and the health care provider may order periodic gentle irrigation with normal saline solution. Because of the danger of perforating the gastric mucosa or disrupting the suture line, the nurse should notify the health care provider if the tube needs to be repositioned or replaced.

What is the most important role of the nurse in preventing rheumatic fever? a. Teach patients with infective endocarditis to adhere to antibiotic prophylaxis. b. Identify patients with valvular heart disease who are at risk for rheumatic fever. c. Encourage the use of antibiotics for treatment of all infections involving a sore throat. d. Promote the early diagnosis and immediate treatment of group A streptococcal pharyngitis.

d. Initial attacks of rheumatic fever and the development of rheumatic heart disease can be prevented by adequate treatment of group A streptococcal pharyngitis. Because streptococcal infection accounts for only about 20% of acute pharyngitis, cultures should be done to identify the organism and direct antibiotic therapy. Viral infections should not be treated with antibiotics. Prophylactic therapy is indicated in those who have valvular heart disease or have had rheumatic heart disease.

When performing a physical assessment of the skin, what should the nurse do first? a. Palpate the temperature of the skin with the fingertips. b. Assess the degree of turgor by pinching the skin on the forearm. c. Inspect specific lesions before performing a general examination of the skin. d. Ask the patient to undress completely so all areas of the skin can be inspected.

d. It is necessary for the patient to be completely undressed for an examination of the skin. Gowns should be provided and exposure minimized as the skin is inspected generally first, followed by a lesion-specific examination. Skin temperature is best assessed with the back of the hand and turgor is best assessed with the skin over the sternum.

When assessing the body functions of a patient with increased ICP, what should the nurse assess first? a. Corneal reflex testing b. Pupillary reaction to light c. Extremity strength testing d. Circulatory and respiratory status

d. Of the body functions that should be assessed in an unconscious patient, cardiopulmonary status is the most vital function and gives priorities to the ABCs (airway, breathing, and circulation).

In instituting a bowel training program for a patient with fecal incontinence, what should the nurse first plan to do? a. Teach the patient to use a perianal pouch. b. Insert a rectal suppository at the same time every morning. c. Place the patient on a bedpan 30 minutes before breakfast. d. Assist the patient to the bathroom at the time of the patient's normal defecation.

d. The first intervention to establish bowel regularity includes promoting bowel evacuation at a regular time each day, preferably by placing the patient on the bedpan, using a bedside commode, or walking the patient to the bathroom. To take advantage of the gastrocolic reflex, an appropriate time is 30 minutes after the first meal of the day or at the patient's usual individual time. Perianal pouches are used to protect the skin only when regularity cannot be established and evacuation suppositories are also used only if other techniques are not successful.

What is the priority intervention in the emergency department for the patient with a stroke? a. Intravenous fluid replacement b. Administration of osmotic diuretics to reduce cerebral edema c. Initiation of hypothermia to decrease the oxygen needs of the brain d. Maintenance of respiratory function with a patent airway and oxygen administration

d. The first priority in acute management of the patient with a stroke is preservation of life. Because the patient with a stroke may be unconscious or have a reduced gag reflex, it is most important to maintain a patent airway for the patient and provide oxygen if respiratory effort is impaired. IV fluid replacement, treatment with osmotic diuretics, and avoiding hyperthermia may be used for further treatment.

Following change-of-shift handoff, which patient should the nurse assess first? a. A 58-year-old male experiencing phantom pain and requesting analgesic b. A 72-year-old male being transferred to a skilled nursing unit following repair of a hip fracture c. A 25-year-old female in left leg skeletal traction asking for the weights to be lifted for a few minutes d. A 68-year-old male with a new lower leg cast complaining that the cast is too tight and he cannot feel his toes

d. The patient with a tight cast may be at risk for neurovascular compromise (impaired circulation and peripheral nerve damage) and should be assessed first. The other patients should be seen as soon as possible. Providing analgesia for the patient with phantom pain would be the next priority. The patient in skeletal traction needs explanation of the purpose and functioning of the traction. She may need analgesia or muscle relaxants to help tolerate the traction.

A patient hospitalized for 1 week with subacute infective endocarditis is afebrile and has no signs of heart damage. Discharge with outpatient antibiotic therapy is planned. During discharge planning with the patient, what is it most important for the nurse to do? a. Plan how his needs will be met while he continues on bed rest. b. Encourage the use of diversional activities to relieve boredom and restlessness. c. Teach the patient to avoid crowds and exposure to upper respiratory infections. d. Assess the patient's home environment in terms of family assistance and hospital access.

d. The patient with outpatient antibiotic therapy requires vigilant home nursing care and it is most important to determine the adequacy of the home environment for successful management of the patient. The patient is at risk for life-threatening complications, such as embolization and pulmonary edema, and must be able to access a hospital if needed. Bed rest will not be necessary for the patient without heart damage. Avoiding infections and planning diversional activities are indicated for the patient but are not the most important factors while he is on outpatient antibiotic therapy.

The patient's diagnosis is a large rectocele requiring surgery. What nursing interventions will be the priority postoperatively? a. An ice pack to relieve swelling b. An enema each day to relieve constipation c. Administration of a stool softener each night d. Perineal care after urination or defecation

d. The primary goal of care is to prevent wound infection and pressure on the vaginal incision, which requires perineal cleansing at least twice daily and after each urination and defecation. An ice pack and stool softener will be used but they are not the priority. The enema would have been done preoperatively.

The patient with advanced cancer is having difficulty controlling her pain. She says she is afraid she will become addicted to the opioids. What is the first thing the nurse should do for this patient? a. Administer a nonsteroidal antiinflammatory drug. b. Assess the patient's vital signs and behavior to determine the medication to use. c. Have the patient keep a pain diary to better assess the patient's potential addiction. d. Obtain a detailed pain history including quality, location, intensity, duration, and type of pain.

d. The priority in pain management is to obtain a comprehensive history of the patient's pain. This will determine the medications most useful for this patient's pain to enable giving the dose that relieves the pain with the fewest side effects. Teaching the patient about the lack of tolerance and addiction associated with effective cancer pain management will also be important for this patient's pain management.


Kaugnay na mga set ng pag-aaral

Chapter 22 - Wireless Networking

View Set

Chapter 15/8: Long-term Care - Provisions and Disclosures

View Set

Collegiate Personal Finance F151 (Ch 1, 2, 4, 5, 6,& 7)- IUS Spring 2020

View Set

Chapter 53, Nursing Management: Female Reproductive and Genital Problems

View Set

Abdominal and Genitourinary Injuries Chapter: 30

View Set

Ch 18 The Cardiovascular System: The Heart HW

View Set

Chapter 18: Nursing Management of the Newborn 5-8

View Set